返回列表 发帖

求高手解决一道题目

48. A researcher discovered that people who have low levels of immune-system activity tend to score much lower on tests of mental health than do people with normal or high immune-system activity. The researcher concluded from this experiment that the immune system protects against mental illness as well as against physical disease



.
The researcher’s conclusion depends on which of the following assumptions?



A. High immune-system activity protects against mental illness better than normal immune-system activity does.



B. Mental illness is similar to physical disease in its effects on body systems.



C. People with high immune-system activity cannot develop mental illness.



D. Mental illness does not cause people’s immune-system activity to decrease.D



E. Psychological treatment of mental illness is not as effective as is medical treatment.

题目不是说immune-system activity 低的人在测试中的成绩低,研究员的结论就是the immune system protects 对于mental illness 和 physical disease 的作用是一样的。题目问研究员的结果是依据下面那个假设的?
那我就觉得结果应该是B呀,为什么答案是D,D是怎么退出来的
收藏 分享

觉得B不对,body system 和immune system范围不同,D起码沾边

TOP

但是D根本没有解释为什么physical disease的作用是一样的呀

TOP

Necessary assumption. Use negation.

If you negate D, mental illness does cause people’s immune-system activity to decrease, then the immune system cannot protect people's mental health. The conclusion of the stimulus falls apart.

P.S. It is common sense that immune system protects physical health.

TOP

谢谢帮助

TOP

去掉not之后是因果倒置的削弱

TOP

返回列表

站长推荐 关闭


美国top10 MBA VIP申请服务

自2003年开始提供 MBA 申请服务以来,保持着90% 以上的成功率,其中Top10 MBA服务成功率更是高达95%


查看